Kết hợp tối đa M với điều kiện G [M] là 2K_2 miễn phí


11

Có điều gì trong tài liệu gần với vấn đề sau:

Cho một đồ thị hai phía với cân bằng bipartition { U , W } , không có tồn tại một kết hợp hoàn hảo M trong G như vậy mà cho mỗi 2 cạnh u 1 w 1 , u 2 w 2M , có một cạnh u 1 w 2 hoặc cạnh u 2 w 1 (hoặc cả hai) trongG(V,E){U,W}MGu1w1,u2w2Mu1w2bạn2w1 ?G

Nói cách khác, có một khớp hoàn hảo sao cho biểu đồ con cảm ứng G [ M ]2 K 2 -free. (Với hai phần cân bằng, ý tôi là | U | = | W | .)MG[M]2K2|Bạn|= =|W|

Điều kiện bổ sung là một cái gì đó giống như một thái cực đối lập được sử dụng trong vấn đề khớp cảm ứng. Một vấn đề khác có thể liên quan là vấn đề tìm kích thước tối đa khớp trong đồ thị lưỡng cực G sao cho sự co lại của các cạnh trong M làm giảm thiểu số cạnh còn lại trong biểu đồ.MGM

Tôi đã kiểm tra danh sách các vấn đề liên quan phù hợp được đưa ra bởi Plummer trong Kết hợp khớp và đỉnh: chúng "khó" đến mức nào? không thành công.

PS: Vấn đề này là trường hợp đặc biệt của vấn đề quyết định này: - Với một , có M khớp tối đa của đồ thị lưỡng cực G sao cho G [ M ]2 K 2 -free và | M | > k . Nếu đồ thị đầu vào được cân bằng bipartite và k = | U | , chúng tôi nhận được vấn đề trên.kNMGG[M]2K2|M|>kk= =|Bạn|

Cảm ơn bạn.


kết hợp hoàn hảo có thể không phải là từ chính xác. Về cơ bản chúng tôi đang hỏi liệu có một kết hợp tối đa có kích thước với tài sản được đề cập. |Bạn|
Cyriac Antony

Theo một nghĩa nào đó, chúng tôi đang yêu cầu một cái gì đó trái ngược với cái được gọi là khớp mạnh. Một kết hợp mạnh trong đồ thị G là một M phù hợp sao cho không có cạnh nào trong G kết nối bất kỳ hai cạnh nào của MMGMGM
Cyriac Antony

Xin lỗi, bởi , ý tôi là biểu đồ con của G được tạo bởi các đỉnh 'trong' MG[M]GM
Cyriac Antony

Câu trả lời:


5

Sự ngạc nhiên! (cho tôi).
Loại khớp này đã được nghiên cứu trong tài liệu. Họ được gọi là kết nối khớp .

Họ đã được Plummer, Stiebitz và Toft giới thiệu trong nghiên cứu về phỏng đoán Hadwiger. Xem chương "Kết nối được kết nối" của Cameron trong cuốn sách "Tối ưu hóa kết hợp - Eureka, bạn thu nhỏ!"

Trạng thái của các kết quả khớp được kết nối trong biểu đồ lưỡng cực (không cần cân bằng) là mở theo hiểu biết tốt nhất của tôi ( tôi sẽ cập nhật ). Phiên bản có trọng số của vấn đề là NP-Complete cho các đồ thị lưỡng cực. Vấn đề là thời gian đa thức có thể giải được đối với các đồ thị lưỡng cực hợp âm.

Cập nhật: vấn đề là NP-đầy đủ cho các đồ thị lưỡng cực cân bằng (nghĩa là vấn đề chính xác được hỏi trong câu hỏi). Điều này đã được chứng minh trong bài báo " Khả năng đa nhiệm: Kết quả độ cứng và các công trình cải tiến " của Alon et al. Họ cũng báo cáo rằng việc tìm kiếm kích thước của một khớp nối lớn nhất là khó có thể xấp xỉ trong một yếu tố của n1-ε trừ khi NP = co-RP.

Ghi chú được thêm trước đó (dành cho những người quan tâm):
" Kết nối khớp trong biểu đồ lưỡng cực hợp âm " của Jobson et al. (doi: https://doi.org/10.1016/j.disopt.2014.06.003 ) và " Kết nối khớp trong các họ đồ thị đặc biệt " của Caragianis (luận án) là hai tài liệu tham khảo đáng chú ý.


1

Có một cách khác để đặt câu hỏi này. Có một khớp hoàn hảo của đồ thị lưỡng cực G cân bằng sao cho mọi cặp cạnh trong M chính xác ở khoảng cách 1 so với nhau trong G không? (Khoảng cách giữa các cạnh ee ' là chiều dài của một con đường ngắn nhất từ một đỉnh của e đến một đỉnh của e ' ).MGMG
ee'ee'

Do đó, điều kiện bổ sung giúp giảm việc tìm một tập hợp các đỉnh từ biểu đồ đường thẳng của G được ghép theo khoảng cách chính xác 2. Do đó, vấn đề tìm một tập hợp các đỉnh có kích thước tối đa ở khoảng cách chính xác 2 từ mỗi khác là một vấn đề ứng cử viên (là một vấn đề gần gũi với vấn đề đang đề cập). Trong bài báo gần đây Về các khía cạnh thuật toán của việc tô màu mạnh (bởi MA Shalu, S. Vijayakumar, S. Devi Yamini và TP Sandhya), họ gọi vấn đề này là mạnh mẽ.L(G)G

Vấn đề thiết lập Stong được biết là NP-đầy đủ trong một số lớp biểu đồ. Tôi không biết trạng thái của nó trên biểu đồ đường của biểu đồ lưỡng cực. Bài báo nói rằng nó hoàn thành NP trên các biểu đồ lưỡng cực. Quan tâm của chúng tôi ở đây sẽ là lớp biểu đồ đường của biểu đồ lưỡng cực.


chỉnh sửa để sửa lỗi; Tôi nghĩ đồ thị dòng của đồ thị lưỡng cực là lưỡng cực. :)
Cyriac Antony

Tôi nghĩ rằng nên có +1 trong định nghĩa của bạn về khoảng cách giữa các cạnh (theo định nghĩa hiện tại, các cạnh của M sẽ ở khoảng cách 1 vì có một cạnh --- một đường dẫn có độ dài 1 --- nối từng cặp cạnh của M, nhưng bạn thực sự có nghĩa là khoảng cách 2).
Florent Foucaud

đã sửa nó thành "các cạnh ... nằm cách nhau 1". Cảm ơn bạn @Florent Foucaud
Cyriac Antony

Điều đó hoạt động, nhưng thật đáng buồn là "khoảng cách các cạnh" của bạn không tương ứng với khoảng cách đỉnh của các đỉnh tương ứng trong biểu đồ đường.
Florent Foucaud

1
Để làm cho mô hình gần với vấn đề của bạn hơn, hãy nhớ rằng một kết hợp tối đa trong biểu đồ tương ứng với một tập độc lập tối đa trong biểu đồ đường của nó. Do đó, trong biểu đồ đường bạn đang tìm kiếm một tập hợp mạnh cũng là một tập độc lập tối đa (đặc biệt, nó cũng phải là một tập hợp thống trị).
Florent Foucaud
Khi sử dụng trang web của chúng tôi, bạn xác nhận rằng bạn đã đọc và hiểu Chính sách cookieChính sách bảo mật của chúng tôi.
Licensed under cc by-sa 3.0 with attribution required.